Неравенства с двумя модулями решение: Неравенство с несколькими модулями | Подготовка к ЕГЭ по математике

2=1} & \end{array}\right. \)
Проанализируем первый график:
Исходная прямая y = x – 1 превращается в ломаную y = |x – 1|, «отражается» в точке (1; 0) в положительную полуплоскость y > 0.
Далее, ломаная y = |x – 1| опускается на 1 вниз y = |x – 1| – 1.
Наконец, области y = |x – 1| – 1 с отрицательными Y снова отражаются в положительную полуплоскость y > 0.
Второй график – окружность с центром (1; 0), радиусом 1.

Получаем три пары решений.
Ответ: {(0; 0) (1; 1) (2; 0)}.

Пример 2. Решите графически систему неравенств:
a) \( \left\{ \begin{array}{ l } \mathrm{|x-2|+|y-4|\leq 2} & \\ \mathrm{y\leq \frac{1}{x-2}+4} & \end{array}\right. \)
|x – 2| + |y – 4| ≤ 2 – внутренняя область и стороны квадрата с точкой пересечения диагоналей (2; 4), длиной диагоналей 4.
\( \mathrm{y\leq \frac{1}{x-2}+4} \) – область под гиперболой с асимптотами x = 2, y = 4.

Решение – точка A(1; 3) и треугольник BCD, заданный системой трех неравенств:

\( \left\{ \begin{array}{ l } \mathrm{x \geq 2} & \\ \mathrm{y \leq 8 — x} & \\ \mathrm{y \geq x} & \end{array}\right. \)

б) \( \left\{ \begin{array}{ l } \mathrm{1\leq |x+y|\leq 3} & \\ \mathrm{2\leq |x-y|\leq 5} & \end{array}\right. \)
Первое неравенство: \( \mathrm{1\leq |x+y|\leq 3\Rightarrow -3\leq x+y\leq -1\cup 1\leq x+y\leq 3\Rightarrow} \)
\( \mathrm{\Rightarrow -x-3\leq y\leq -x-1\cup -x+1\leq y\leq -x+3} \) – две полосы, параллельные y = –x.
Второе неравенство: \( \mathrm{2\leq |x-y|\leq 5\Rightarrow -5\leq x-y\leq -2\cup 2\leq x-y\leq 5\Rightarrow} \)
\( \mathrm{\Rightarrow -x-5\leq -y\leq -x-2\cup -x+2\leq -y\leq -x+5\Rightarrow} \)
\( \mathrm{\Rightarrow x+2\leq y\leq x+5\cup x-5\leq y\leq x-2} \) – две полосы, параллельные y = x.

Решение – четыре прямоугольника, образованные пересечением полос.
\( \left\{ \begin{array}{ l } \mathrm{-x-3\leq y\leq -x-1\cup -x+1\leq y \leq -x+3} & \\ \mathrm{x+2\leq y\leq x+5\cup x-5\leq y\leq x -2} & \end{array}\right. \)

Пример 3. Найдите значения параметра a, при которых система имеет ровно два решения:
\( \left\{ \begin{array}{ l } \mathrm{y=x^2-5|x|+4} & \\ \mathrm{y\leq a} & \end{array}\right. \)
y = x2 – 5|x| + 4 – парабола y = x2 – 5x + 4 = (x – 1)(x – 4), x > 0, отраженная в отрицательную полуплоскость x < 0 относительно оси Y.
Осью симметрии параболы при x > 0 является прямая \( \mathrm{x_0=\frac{x_1+x_2}{2}=\frac{1+4}{2}=2,5} \)
Вершина лежит на оси. Ордината вершины: y0 = 2,52 – 5 · 2,5 + 4 = –2,25.
В полуплоскости x < 0 вершина расположена симметрично относительно оси Y и имеет ту же ординату.
Значит, при a = –2,25 система имеет два решения:

При a < –2,25 решений нет, при a > –2,25 решений бесконечное множество (отрезки кривой).
Ответ: a = –2,25.

Неравенства с модулем — что это, определение и ответ

Изобразим график функции \(y = |x|\) и несколько прямых, параллельных оси Ох.

Модуль больше отрицательного числа. Модуль меньше отрицательного числа.

Глядя на график, легко убедиться, что если неравенство имеет вид \(\left| x \right| > — 1\) , то его решением будет любое число.

В тоже время неравенство \(\left| x \right| < — 1\) решение иметь не будет, так как неотрицательное число не может быть меньше отрицательного.

Модуль больше положительного числа. Модуль меньше положительного числа.

Теперь сравним модуль с положительным числом. Рассмотрим такой пример: \(\left| x \right| < 1\). На графике это соответствует нижней части «уголка».

Раскроем модуль как обычно.

На положительном промежутке числовой прямой раскрываем модуль с изначальным знаком:

\(\left\{ \begin{matrix} \ \\ x \geq 0 \\ x < 1 \\ \ \\ \end{matrix} \right.\ \) \(\rightarrow x \in \lbrack 0;1)\)

На отрицательном промежутке числовой прямой раскрываем модуль с противоположным знаком:

\(\left\{ \begin{matrix} \ \\ x < 0 \\ — x < 1 \\ \ \\ \end{matrix} \right.\ \)

Имеем \(\left\{ \begin{matrix} \ \\ x < 0 \\ x > — 1 \\ \ \\ \end{matrix} \right.\ \) \(\rightarrow x \in ( — 1;0)\).

Мы рассматривали 2 случая, то есть формально получили совокупность двух систем.

\(\left\lbrack \begin{matrix} \ \\ \left\{ \begin{matrix} \ \\ x \geq 0 \\ x < 1 \\ \ \\ \end{matrix} \right.\ \\ \left\{ \begin{matrix} \ \\ x < 0 \\ x > — 1 \\ \ \\ \end{matrix} \right.\ \\ \ \\ \end{matrix} \right.\ \)

Значит, решения, полученные в каждом случае, необходимо объединить.

Получим, что \(x \in ( — 1;1)\)

В общем виде решение неравенства, вида \(\left| f\left( x \right) \right| < a\) будет иметь вид:

\(\left| f\left( x \right) \right| < a \Longleftrightarrow \left\lbrack \begin{matrix} \ \\ \left\{ \begin{matrix} \ \\ f\left( x \right) < a \\ f(x) \geq 0 \\ \ \\ \end{matrix} \right.\ \\ \left\{ \begin{matrix} \ \\ — f\left( x \right) < a \\ f\left( x \right) < 0 \\ \ \\ \end{matrix} \right.\ \\ \ \\ \end{matrix} \right.\ \)

\(\left\lbrack \begin{matrix} \ \\ \left\{ \begin{matrix} \ \\ f\left( x \right) < a \\ f(x) \geq 0 \\ \ \\ \end{matrix} \right.\ \\ \left\{ \begin{matrix} \ \\ f\left( x \right) > — a \\ f\left( x \right) < 0 \\ \ \\ \end{matrix} \right. \ \\ \ \\ \end{matrix} \right.\ \Longleftrightarrow — a < f\left( x \right) < a\)

Более кратко имеем:

\(|f\left( x \right)| \leq a \Longleftrightarrow — a < f\left( x \right) < a\)

Теперь давайте перейдем к неравенству вида \(\left| x \right| > 1\). На графике ему соответствуют «рожки». Раскроем модули для каждого случая.

На положительном промежутке числовой прямой раскрываем модуль с изначальным знаком

\(\left\{ \begin{matrix} \ \\ x \geq 0 \\ x > 1 \\ \ \\ \end{matrix} \right.\ \rightarrow x \in (1; + \infty)\)

На отрицательном промежутке числовой прямой раскрываем модуль с противоположным знаком:

\(\left\{ \begin{matrix} \ \\ x < 0 \\ — x > 1 \\ \ \\ \end{matrix} \right.\ \) Имеем \(\left\{ \begin{matrix} \ \\ x < 0 \\ x < — 1 \\ \ \\ \end{matrix} \right.\ \rightarrow x \in ( — \infty;1)\)

Теперь нам опять оба случая необходимо объединить совокупностью и затем объединить решения.

\(\left\lbrack \begin{matrix} \ \\ \left\{ \begin{matrix} \ \\ x \geq 0 \\ x > 1 \\ \ \\ \end{matrix} \right. \ \\ \left\{ \begin{matrix} \ \\ x < 0 \\ x < — 1 \\ \ \\ \end{matrix} \right.\ \\ \ \\ \end{matrix} \right.\ \)

Тогда \(x \in ( — \infty;1) \cup (1; + \infty)\).

Этот результат соответствует тому, что видно на графике.

В общем виде решение неравенства, вида \(\left| f\left( x \right) \right| > a\) будет иметь вид:

\(\left| f\left( x \right) \right| > a \Longleftrightarrow \left\lbrack \begin{matrix} \text{\ \ } \\ \left\{ \begin{matrix} \ \\ f\left( x \right) > a \\ f(x) \geq 0 \\ \ \\ \end{matrix} \right.\ \\ \left\{ \begin{matrix} \ \\ — f\left( x \right) > a \\ f\left( x \right) < 0 \\ \ \\ \end{matrix} \right.\ \\ \ \\ \end{matrix} \right.\ \)

\(\left\lbrack \begin{matrix} \ \\ \left\{ \begin{matrix} \ \\ f\left( x \right) > a \\ f(x) \geq 0 \\ \ \\ \end{matrix} \right.\ \\ \left\{ \begin{matrix} \ \\ f\left( x \right) < — a \\ f\left( x \right) < 0 \\ \ \\ \end{matrix} \right.\ \\ \ \\ \end{matrix} \right.\ \Longleftrightarrow \left\lbrack \begin{matrix} \ \\ f\left( x \right) > a \\ f\left( x \right) < — a \\ \ \\ \end{matrix} \right.

\ \)

Более кратко имеем:

\(\left| f\left( x \right) \right| > a \Longleftrightarrow \left\lbrack \begin{matrix} \ \\ f\left( x \right) > a \\ f\left( x \right) < — a \\ \ \\ \end{matrix} \right.\ \)

Несколько модулей

Неравенство может так же содержать несколько модулей.

\(\left| f\left( x \right) \right| + \left| g\left( x \right) \right| + \ldots + \left| p\left( x \right) \right| < a\) или \(\left| f\left( x \right) \right| + \left| g\left( x \right) \right| + \ldots + \left| p\left( x \right) \right| > a\).

Для решения такого вида неравенств следует воспользоваться алгоритмом:

  1. Определить критические точки и разделить прямую на промежутки;

  2. В каждом из промежутков раскрыть модуль с соответствующим знаком;

  3. Для каждого случая решить систему неравенств;

  4. Объединить полученные результаты.

Пример.

\(\left| x + 3 \right| + \left| 2x — 1 \right| > 5\)

  1. Определим критические точки:

\(x + 3 = 0 \rightarrow x = — 3\)

\(2x — 1 = 0 \rightarrow x = 0,5\)

Таким образом имеем 3 промежутка: \(x \in \left( — \infty; — 3 \right\rbrack;x \in \left( — 3;0,5 \right\rbrack;x \in (0,5;\ + \infty)\).

  1. \(\left\lbrack \begin{matrix} \ \\ \left\{ \begin{matrix} \ \\ x \in ( — \infty; — 3\rbrack \\ — x — 3 — 2x + 1 > 5 \\ \ \\ \end{matrix} \right.\ \\ \left\{ \begin{matrix} \ \\ x \in ( — 3;0,5\rbrack \\ x + 3 — 2x + 1 > 5 \\ \ \\ \end{matrix} \right.\ \\ \left\{ \begin{matrix} \ \\ x \in (0,5;\ + \infty) \\ x + 3 + 2x — 1 > 5 \\ \ \\ \end{matrix} \right.\ \\ \ \\ \end{matrix} \right.\ \)

  2. Решим каждую из полученных систем:

\(\left\lbrack \begin{matrix} \ \\ \left\{ \begin{matrix} \ \\ x \in ( — \infty;\ — 3\rbrack \\ — 3x > 7 \\ \ \\ \end{matrix} \right.\ \\ \left\{ \begin{matrix} \ \\ x \in ( — 3;0,5\rbrack \\ — x > 1 \\ \ \\ \end{matrix} \right.\ \\ \left\{ \begin{matrix} \ \\ x \in (0,5;\ + \infty) \\ 3x > 3 \\ \ \\ \end{matrix} \right.\ \\ \ \\ \end{matrix} \right.\ \ \rightarrow \left\lbrack \begin{matrix} \ \\ \left\{ \begin{matrix} \ \\ x \in ( — \infty;\ — 3\rbrack \\ x < \frac{7}{3} \\ \ \\ \end{matrix} \right. {2} — 2x + 1\)

\(6x + 2x < 1 — 9\)

\(8x > — 8\)

\(x > — 1\)

Предварительное исчисление алгебры — Неравенство с двумя абсолютными значениями

спросил

Изменено 3 года, 5 месяцев назад

Просмотрено 32к раз

$\begingroup$

Я здесь новенький, и мне было интересно, может ли кто-нибудь из вас помочь мне с этой маленькой проблемой, которая уже действует мне на нервы, так как я пытался решить ее в течение нескольких часов.

Готовясь к очередному тесту на неравенства с абсолютными значениями, я нашел вот этот:

$$ |x-3|-|x-4|

Вот неравенства: $$ x−3 < x−4 +x $$ $$ x−3 < −(x−4) +x $$ $$ −(x−3)<−(x−4)+x $$

И вот соответственно мои ответы: $$ x>1, \quad x>-1, \quad x<7 $$

Я буду очень признателен, если кто-нибудь сможет мне помочь, потому что я уже устал решать эту проблему, которая, кстати, не требует, чтобы я ее разгадал, но знаете, почему бы и нет?

  • алгебра-предварительное исчисление
  • неравенство
  • абсолютное значение

$\endgroup$

3

$\begingroup$

Как решить $|x-3|-|x-4|

Я бы начал с того, что $|x-3|=x-3$ для $x \geq 3$, тогда как $|x-3 |=3-x$ для $x \leq 3$. Аналогично, $|x-4|=x-4$ для $x \geq 4$, тогда как $|x-4|=4-x$ для $x \leq 4$

  1. , если $x \geq 4$ тогда $x > 3$, следовательно, $f(x)=x-3-(x-4)=1$ и мы находим $1

  2. , если $x \leq 3$, то $x < 4$, следовательно, $f(x)=3-x-(4-x)=-1$ и мы имеем $-1

  3. если $3

Складываем все это вместе, получаем $(-1,3] \cup (3,4) \cup [4,\infty) = (-1, \infty)$, что и является вашим ответом. Обычно я решаю такие вещи с помощью метода знаков.

$\endgroup$

$\begingroup$

То, что у вас есть, почти правильно, последний шаг — ограничить ваше решение соответствующим регионом.

Например, для $x>1$ ответ должен быть в области $x>4$, поэтому ваш ответ для этой области — $x>4$.

Для $x>-1$ ваш ответ должен быть в регионе $x<3$, поэтому ваш ответ для этого региона $-1

И для последнего, $x<7$, ваш ответ должен быть в соответствующем регионе, который у нас был первым, а именно $3

Теперь нарисуйте эти три ответа на числовой прямой, и вы получите $x>-1$, желаемый окончательный ответ.

$\endgroup$

11

$\begingroup$

Стандартный полумеханический способ устранения знаков абсолютного значения состоит в том, чтобы разделить числовую прямую на сегменты. Критическая точка для $|x-4|$ находится при $x=4$, а критическая точка для $|x-3|$ — при $x=3$.

Предположим сначала, что $x \ge 4$. Тогда $|x-4|=x-4$ и $|x-3|=x-3$. Итак, мы имеем дело с неравенством $(x-3)-(x-4)\lt x$, то есть с $1\lt x$, что заведомо верно при $x\ge 4$.

Теперь предположим, что $3\le x\lt 4$. Тогда $|x-4|=4-x$ и $|x=3|=x-3$, поэтому мы рассматриваем неравенство $(x-3)-(4-x)\lt x$, что есть $2x-7\lt x$. Это упрощается до $x\lt 7$, что, безусловно, равно истинным в интервале $[3,4)$.

Вероятно, в этот момент ваш расчет сбился с пути. Мы смотрели на интервал $[3,4)$ и спрашивали, какие точки в этом интервале удовлетворяют нашему неравенству. Обработка показала нам, что все точки этого интервала удовлетворяют $x\lt7$. Ну, они все делают!

Наконец, предположим, что $x\lt 3$. Тогда $|x-4|=4-x$ и $|x-3|=3-x$. Итак, мы рассматриваем неравенство $(3-x)-(4-x)\lt x$. Рассчитать. Левая часть равна $-1$, поэтому для интервала $(-\infty,3)$ неравенство выполняется именно при $-1\lt x$.

Сопоставив все вместе, мы заключаем, что исходное неравенство выполняется (i) если $x\ge 4$; (ii) если $3\le x\lt 4$; и (ii) если $x\lt 3$, но $-1\lt x$. Этот сложный набор условий можно обобщить гораздо проще: $x\gt -1$.

Существуют и другие способы описания набора решений. Например, мы могли бы сказать, что набор равен $(-1,\infty)$.

$\endgroup$

4

$\begingroup$

Вот один из способов сделать это.

Если $x\ge4$, то $|x-3|=x-3$ и $|x-4|=x-4$, поэтому нужно $(x-3)-(x-4)\ lt x$, которое вы должны быть в состоянии решить (не забывая проверить ваше решение на соответствие предположению $x\ge4$.

Если $3\le x\le4$, то $|x-3|=x-3$ и $|x-4|=4-x$, так что вы получите ….

Если $x\le3 $, затем $|x-3|=\dots$

$\endgroup$

$\begingroup$

$|x-3|-|x-4|< x$, я пишу $|x-3|< x+|x-4|$

, но помните: $|r|< s \ подразумевает -s < г < с$.

Итак, я записываю уравнение в виде: $-x-|x-4| < х-3 < х+|х-4|$.

Из этого неравенства я получаю 2 уравнения:

(a) $-x-|x-4| < х-3$

(б) $x-3 < x+|x-4|$.

Помните также: $|r| > s \ подразумевает r > s \text{ или } r < -s $.

Так что эту концепцию я буду применять к уравнению (а) и уравнению (б).

Из уравнения (а):

$-x-|x-4| < x-3$ Я пишу так, что абсолютное значение находится в одной стороне: -|x-4| < 2x-3, то я умножаю на -1: |x-4| > -2x+3 . Теперь я записываю уравнение в виде: |r| > s—-> r > s ИЛИ r < -s . В результате получаем 2 дополнительных уравнения: x-4 > -2x+3 ИЛИ x-4 < 2x-3 . Из первого: 3x > 7 —>x > 7/3 или (7/3,∞) . Из второго неравенства: -1 < x или (-1,∞). Первое и второе неравенства являются функцией ИЛИ, тогда: (7/3,∞) U (-1,∞) равно: (-1,∞)

Теперь уравнение (b): x-3 < x+|x-4| мы пишем, чтобы поместить абсолютное значение в одну сторону: $-3 < |x-4|$ или $|x-4| > -3$ . Для выполнения этого неравенства $x$ будет принимать любое положительное или отрицательное значение. В результате мы можем записать значение $x$ для уравнения (b) в виде: $(-∞,+∞)$

Конечным результатом уравнения (a) И (b) будет пересечение их значение: $(a)∩(b)$ или $(-1,∞)∩(-∞,∞)$ и найти окончательный результат для $x: (-1,∞)$, удовлетворяющий неравенству $| х-3|-|х-4| < х $

$\endgroup$

$\begingroup$

для /x-3/-/x-4/

1-й, если оба абсолютных значения положительны. x=+1

2-й, если оба отрицательные, x=-1

3-й, если 1-й отрицательный, а 2-й положительный x=7/3

4-й, если 1-й положительный. и 2-й отрицательный. х=7. поэтому все эти числа дают нам решение, кроме -1, поэтому все числа выше -1 включены в решение. или S=x>-1. или от -1 до бесконечности.

$\endgroup$

Неравенства абсолютного значения | Brilliant Math & Science Wiki

Вспомним определение абсолютного значения:

Для любого действительного числа \(x\) его абсолютное значение определяется как

.

\[|х| = \begin{case} x & \textrm{if } x > 0 \\ 0 & \textrm{если} х = 0 \\ -x & \textrm{if} x < 0. \\ \end{case}\]

Функция \(f(x) = |x| \) также называется функцией модуля. \(_\квадрат\)

Пусть \(x\) будет переменной или алгебраическим выражением, и пусть \(a\) будет вещественным числом таким, что \(a > 0\). Тогда выполняются следующие неравенства:

  1. \(|x| \leq a \Leftrightarrow -a \leq x \leq a\)
  2. \(|x| \geq a \Leftrightarrow x \leq -a\ \) или \(\ x \geq a\)
  3. \(|x| < a \Leftrightarrow -a
  4. \(|x| > a \Leftrightarrow x <-a\ \) или \(\ x>a\).

Для функций \(f(x)\) и \(g(x),\) от \(x,\) имеем

\(1.~~ |f(x)| \leq g(x )~~ \Leftrightarrow~~ — g(x) \leq f(x) \leq g(x)\\
2.~~|f(x)|\geq g(x) ~~ \Leftrightarrow~~ f (x) \leq -g(x) \text{ или }f(x) \geq g(x)\\
3.~~|f(x)|< g(x) ~~ \Leftrightarrow ~~- ~g(x) 4.~~|f(x)| > g(x) ~~ \Leftrightarrow ~~f(x) <- g(x) \ text{ или }f(x)>g(x). \)

Вот некоторые другие свойства абсолютного значения в неравенствах:

  1. \(|a+b|\leq |a|+|b|\) с равенством, если оба имеют одинаковый знак, т. е. \(ab > 0\).
  2. \(|a-b|\leq |a|+|b|\) с равенством, если они имеют разные знаки, т. е. \(ab<0\).

Обобщая два приведенных выше неравенства, получаем

\[|\pm a_1 \pm a_2 \pm a_3 \pm \cdots \pm a_n| \leq |a_1| +|a_2|+\cdots +|a_n|.\]

Давайте рассмотрим пример применения этой формулы.

Найдите минимальное значение \(|x-1|+|x-2|+\dots + |x-100|\).


Используя приведенное выше неравенство, мы пытаемся удалить переменную \(x\) на

\[\ начало {выровнено} |x-1|+|x-2|+\dots + |x-100| &\geq |(x-1)+(x-2)+\dots +(x-50)-(x-51)-(x-52)-\dots -(x-100)| \\ &= 2500. \конец{выровнено}\]

Чтобы убедиться, что это допустимый случай, мы должны иметь \(\{(x-1), (x-2), \dots , (x-50)\}\geq 0\) и \(\{( x-51), (x-52), \dots , (x-100)\}\leq 0\). Это сводится только к двум случаям:

\[\begin{случаи} х-50 &\geq 0 \\ х-51 и \leq 0. \end{случаи}\]

Следовательно, минимальное значение получается при \(x\in [50,51]\). \(_\квадрат\)

Вот задача.

\[х={-12}, у={14} \] \[х={-12}, у={12} \] \[х={-13}, у={13} \] \[ x={-13}, y={14} \]

Если множество решений системы неравенств \[\begin{case}\lvert a+1 \rvert < 3\\ \lvert b-1 \rvert <10 \конец{случаи}\] есть \( x < a+b < y, \), то что такое \(x\) и \(y?\)

Здесь мы рассмотрим определение получения абсолютного значения числа. Чтобы «отменить» знаки абсолютного значения, мы могли бы получить положительное или отрицательное значение, поскольку абсолютное значение \( — 5 \) совпадает с абсолютным значением \( 5 \), которое равно \( 5\). Это становится методом, в котором у нас есть несколько случаев.

Основные шаги (для работы с линейными/множественными линейными абсолютными неравенствами):

  1. «Отменить» знаки абсолютного значения, сделав выражения внутри знака абсолютного значения отрицательными или положительными.
  2. Возьмите все полученные неравенства (это все множества решений) и вычислите множество решений. Чтобы вычислить набор решений, учитывая набор наборов решений из проработанных вами случаев, рассмотрите исходную задачу как кусочную функцию, поэтому у вас есть случаи. При каких значениях \(x\) левая часть становится отрицательной? Положительный? Нуль? Все это дополнительные ограничения на \(x\), и вы возьмете точку пересечения этих дополнительных ограничений на \(x\) и «окончательное» неравенство, полученное в результате проделанной вами работы. Это дает то, что я называю «ограниченным окончательным» неравенством.
  3. Наконец, вы берете объединение всех «ограниченных окончательных» неравенств для вашего окончательного набора решений.

Мы рассмотрим, как это сделать, в следующих трех примерах.

Решите \( | x + 3 | < 7 \).


Случай 1: \(x+3\) неотрицательно, или \(x\geqslant -3\)
Свойство абсолютного значения говорит нам, что \(|a| = a\) для не- отрицательный \(a\), так что в этом случае \[|x+3|<7 \подразумевается x+3 < 7\подразумевается x<4. \] Здесь у нас есть два неравенства, и решением для этого случая является пересечение обоих неравенств. Это потому, что \(x\) должно удовлетворять обоим, поскольку они зависят друг от друга \((\) только потому, что \(x\geq -3\) у ​​нас есть \(x<4)\). Следовательно, решение для этого случая равно \(-3 \leq x < 4\).

Случай 2: \(x+3\) отрицательно, или \(x < -3\)
Опять же, \(|a| = -a\) для отрицательного значения \(a\), поэтому \[|x+3|<7\подразумевается -(x+3)<7\подразумевается x>-10.\] По той же причине, что и выше, мы должны взять пересечение обоих неравенств, что равно \(-10

Наконец, возьмем объединение этих неравенств, так как они независимы друг от друга: \[-10

Решите \( 2|x+2| — |x+5| \leqslant 4 \).


Случай 1: \( x+2>0 \text{ и } x+5>0 \подразумевает x>-2\)
В этом случае имеем \[\начать{выравнивать} 2(х+2) — (х+5) &\leqslant 4 \\ 2x + 4 — x — 5 &\leqslant 4\\ х -1 &\leqslant 4 \\ х &\leqslant 5\\ \Стрелка вправо -2 \конец{выравнивание}\]

Случай 2: \( x+2>0 \text{ и } x+5\leqslant 0 \)
Поскольку всегда верно, что \(x+5>x+2,\), этот случай невозможен .

Случай 3: \( x+2\leqslant 0 \text{ и } x+ 5>0 \подразумевается -5 В этом случае имеем \[\начать{выравнивать} -2(x+2) — (x+5) &\leqslant 4 \\ -2x — 4 — x — 5 &\leqslant 4\\ -3x — 9 &\leqslant 4\\ -3x &\leqslant 13\\ х &\geqslant \dfrac{-13}{3}\\ \Rightarrow \dfrac{-13}{3} \leqslant x &\leqslant -2. \конец{выравнивание}\]

Случай 4: \( x+2\leqslant \text{ и } x + 5\leqslant \подразумевает x\leqslant -5\)
В этом случае имеем \[\начать{выравнивать} -2(x+2) + (x+5) &\leqslant 4 \\ -2x — 4 + x + 5 &\leqslant 4\\ -x +1 &\leqslant 4 \\ -x &\leqslant 3 \\ х &\geqslant 3, \конец{выравнивание} \] что невозможно, так как \(x\leqslant -5.\) 92 — \dfrac{1}{2} &< 0 \\ \left( x+ 1 + \dfrac{1}{\sqrt{2}} \right) \left( x + 1 - \dfrac{1}{\sqrt{2}}\right) &< 0. \конец{выравнивание} \] Чтобы решить это квадратное неравенство, мы можем использовать диаграмму анализа знаков.

Мы знаем, что неравенство будет равно нулю, когда один множитель всего выражения равен нулю, а именно при \( -1 — \frac{1}{\sqrt{2}} \) и \( -1 + \frac{1 }{\sqrt{2}} \), поэтому мы хотим знать, является ли каждый фактор положительным или отрицательным при определенных значениях \( x \), которые меньше, чем \( -1 — \frac{1}{\sqrt{ 2}} \), больше \( -1 — \frac{1}{\sqrt{2}} \) и меньше \(-1 + \frac{1}{\sqrt{2}} \) и больше -1 + \( \frac{1}{\sqrt{2}} \). Мы также используем свойства, согласно которым отрицательное значение, умноженное на отрицательное, дает положительное значение, положительное значение, умноженное на положительное, дает положительное значение, а отрицательное значение, умноженное на положительное, дает отрицательное значение. Чтобы убедиться, что левая часть меньше нуля, мы ищем \(x \), которые дают нам отрицательные значения левой части.

Мы видим, что левая часть квадратного неравенства меньше нуля, когда \( -1 — \frac{1}{\sqrt{2} } < x < -1 + \frac{1}{\sqrt{ 2}}.\qquad (1) \) 92 - \dfrac{1}{2} &< 0 \\ \left( x+ 3 + \dfrac{1}{\sqrt{2}} \right) \left( x + 3 - \dfrac{1}{\sqrt{2}}\right) &< 0. \конец{выравнивание} \] Снова делаем таблицу анализа знаков:

Мы видим, что левая часть квадратного неравенства меньше нуля, когда \( -3 — \frac{1}{\sqrt{2} } < x < -3 + \frac{1}{\sqrt{ 2}}.\qquad (2) \)
\((\)Заметим, что эти неравенства удовлетворяют \(x < -2.)\)

Теперь у нас есть два составных неравенства (1) и (2), оба из которых являются решениями; \(x\) может лежать в любом интервале, поэтому окончательное решение равно

\[-3 — \dfrac{1}{\sqrt{2}} < x < -3 + \dfrac{1}{\sqrt{2}}\ \text{ или }\ -1 - \dfrac{1 }{\sqrt{2}} < x < -1 + \dfrac{1}{\sqrt{2}}.\ _\square\]

8 -16 Ни один из других вариантов 16

Для всех действительных чисел \(x\), каково максимальное значение

\[|3x+8|-|4x| ?\]

\[ Y = |x| — |х + 1| + |х + 2| — |х + 3| + \cdots +|x + 2016| \]

Найдите минимальное значение \( Y \).

\[\] Обозначение : \( | \cdot | \) обозначает функцию абсолютного значения.

Еще один способ решить абсолютные неравенства — изобразить их в виде графика.

Решите \( | x + 3 | < 7 \).


Решение с графом состоит в том, чтобы выяснить, когда \( y= |x+3| \) меньше, чем \( y = 7 \).

Чтобы нарисовать первый график, мы знаем, что на основе определения абсолютного значения \( |x+3| = \begin{cases} x+3 & \textrm{if } x > -3 \\ 0 & \textrm{если} х = -3 \\ -x-3 & \textrm{if } x < -3. \\ \end{case} \)

Здесь мы видим, что когда \( \boxed{-10 < x < 4} \), \( y = |x+3| \) меньше, чем \( y = 7 \). Мы используем открытые знаки меньше, потому что если \( x = -10 \) или \( x = 4 \), то \( y = |x+3| \) будет равно \( y = 7 \), но исходное неравенство задается, когда \( |x+3| \) меньше, чем \( 7 \), не меньше или равно \( 7 \). \(_\квадрат\)

Решите \( 2|x+2| — |x+5| < 4 \).


Мы также можем решить это графически. Но нам нужно будет использовать определение абсолютного значения, чтобы написать кусочную функцию для \(y = 2|x+2| — |x+5| \).

По падежам получаем следующее:

  • Если \( |x+2| \) и \( |x+5| \) оба положительны, результирующее уравнение имеет вид \( y = x-1 \).
  • Если \( |x+2| \) отрицательное, а \( |x+5| \) положительное, результирующее уравнение будет \(y= -3x — 9 \).
  • Если \( |x+2| \) положительное, а \( |x+5| \) отрицательное, результирующее уравнение будет \( y= 3x+9 \).
  • Если \( |x+2| \) и \( |x+5| \) оба отрицательны, результирующее уравнение будет \( y = -x+1 \).

Давайте тогда начнем рассматривать некоторые наборы значений \(x\).

  • Если \( x < -5 \), то мы знаем, что и \( |x+2 | \), и \( |x+5| \) будут отрицательными. Таким образом, когда \( x < -5 \), \ ( y = -x + 1 \).
  • Если \( -5 < x < -2 \), то мы знаем, что \( |x+2| \) будет отрицательным, а \( |x+5| \) будет положительным. Таким образом, когда \(-5 < x < -2 \), \(y = -3x - 9 \).
  • Если \( x > -2 \), то мы знаем, что и \( |x+2| \), и \( |x+5| \) будут положительными. Таким образом, при \( x > -2 \), \( y = x -1 \).
  • Вы можете вычислить значения \( y \) в точках \( x = -5 \) и \( x = -2 \), так как они являются пересечением \( y = -x + 1 \) и \( y = -3x — 9 \) и пересечение \( y = -3x — 9 \) и \( y = x-1 \) соответственно.

Затем просто постройте график \( y = 4 \) и определите, когда \( y = 2|x+2| — |x+5 | \) меньше, чем \( y = 4 \).

Получаем следующий график:

Примечание. Мой график просто уменьшен, чтобы показать точки пересечения двух графиков. 92 + 4x + 4| — 2|х+2| + 1 \) меньше, чем \( \dfrac{1}{2} \), когда

\[ \boxed{ -3 — \dfrac{1}{\sqrt{2}} < x < -3 + \dfrac{1}{\sqrt{2}} \quad \text{or} \quad -1 - \dfrac{1}{\sqrt{2}} < x < -1 + \dfrac{1}{\sqrt{2}}}. \]

На самом деле мы этого не видим, поэтому вам придется решить некоторые уравнения, чтобы получить эти точные значения, используя сгенерированную нами кусочную функцию и \( y = \dfrac{1}{2} \), но это помогает иметь график, потому что вам не придется преодолевать столько проблем, как метод случаев. \(_\квадрат\)

Докажите, что нет решения для \(|3x+4|+|2x+3|+|x+2| = \frac{1}{2}\).


\(\text{LHS}\) этого уравнения представляет собой сумму трех линейных членов абсолютных значений. Это означает, что \(\frac{1}{2}\) меньше минимального значения, которого можно достичь, скажем, \(k\). Это мотивирует нас найти минимальное значение выражения. Хорошим инструментом было бы неравенство

\[|a|+|b|\geq |\pm a\pm b|.\]

Существует слишком много возможных способов присваивать знаки терминам. Давайте еще раз посмотрим на выражения. Чтобы лучше понять минимальное значение, мы пытаемся удалить все переменные, чтобы результат был константой. Нетрудно видеть, что возможны два пути:

\[\begin{выравнивание} |3x+4|+|2x+3|+|x+2|&\geq |-(3x+4)+(2x+3)+(x+2)|\\ |3x+4|+|2x+3|+|x+2|&\geq |+(3x+4)-(2x+3)-(x+2)|. \конец{выравнивание}\]

Пробуем первый способ:

\[\ начало {выровнено} |3x+4|+|2x+3|+|x+2| & \geq |-(3x+4)+(2x+3)+(x+2)| \\ & = 1. \конец{выровнено}\]

Это означает, что минимальное значение равно \(1,\) и, поскольку \(\frac{1}{2} < 1\), решение приведенного выше уравнения отсутствует.

Обратите внимание, что мы всегда должны проверять правильность неравенства. В этом случае имеем систему неравенств 92 — 6у + 5 \leq 0 \). Факторизация дает \((y-1)(y-5) \leq 0 \ подразумевает 1 \leq y \leq 5 \). Это означает, что \( 1 \leq |x| \leq 5 \), и, следовательно, решения равны \( -5 \leq x \leq -1 \) и \( 1 \leq x \leq 5 \). \(_\квадрат\)

Найдите решения неравенства \( |x+1| + |x+2| + |x+3| \leq -1.\)


Вместо того, чтобы пробовать варианты этой задачи, обратите внимание, что результат взятия абсолютного значения всегда дает положительное значение (или ноль), а сумма трех неотрицательных значений не может быть отрицательной. Поскольку \( |x+1| + |x+2| + |x+3| \) должно быть неотрицательным, оно никогда не будет меньше \(-1 \). Таким образом, имеется нет решения .

Добавить комментарий

Ваш адрес email не будет опубликован. Обязательные поля помечены *